Establishing a Business (Partnerships, Corporations, etc.)

Réussis tes devoirs et examens dès maintenant avec Quizwiz!

. Chaco and Howard are both general partners at a business firm, Ham, Ham, McGoo (HHM), which is a Limited Partnership (LP). Kim is a limited partner that works for HHM, so she does not participate in day-to-day management decisions or otherwise run the business. Jamie and Everette are both employees who work in the mailroom. In the midst of a financial crisis, HHM was forced to declare bankruptcy. Who will have his/her/their personal assets seized by the creditors of HHM? A. Jamie and Everette B. Howard and Chaco C. Kim, but not Howard and Chaco, because general partners get preferential treatment D. Everyone who works at the firm (Jamie, Everette, Howard, Chaco, and Kim) E. Howard, Chaco, and Kim, because they are all partners

- B. Topical Lecture; Tug-of-War Example in Topical Talk on Partnerships. LSB 144; Barron's pages 342-343. General partners (Howard and Chaco) equally share in both the risk of losses and the potential for gains (some form of agreement, implied or express) - "one for all, and all for one." A limited partner (Kim) only risks what he/she puts into the business and is not held personally liable for partnership obligations beyond his/her capital contribution.

. Leo, Raphael, and Mike open up a pizza restaurant as a general partnership. The business is operating well. One day, Leo examines the business records and realizes that Mike authorized a loan in the business's name. The loan had not been repaid and the amount of interest had grown quickly. Leo informed Raphael, and Raphael dissociated from the partnership. When the loan holder brought a claim, Raphael was included as a defendant. Is Raphael liable? a. Yes, because the loan was taken out while he was a partner. b. Yes, but only to the extent of his initial investment in the partnership. c. No, because he left before the lawsuit was filed. d. No, because dissociating partners are not liable for the debts of the partnership. e. No, because the loan was executed without his knowledge.

A. Barron's page 339 (specifically): LSB 140 (generally) Departing partners are liable for partnership obligations existing at the time of their departure. Since the loan was in existence while Raphael was a partner, he will be liable. He is a general partner since no agreement or paperwork was made to the contrary. Answer B would be true if Raphael was a limited partner in the business at the time of the loan. Answers C, D, and E are false due to previous explanation.

. In a limited partnership, which partners are most protected from personal liability? a. General partners with the least financial investment b. All partners have equal protection c. Non-decision making limited partners d. Decision making shielded partners e. Decision making limited partners

C. Lectures; LSB 144. Limited partners have no management or control of the business "and are not held personally liable for partnership obligations beyond the amount of his/her capital contribution.

. Choose the False statement: • a. While some federal laws regulate corporate activities, corporations are generally regulated at the state level. • b. Vince and Ari were in a partnership, but they now want to end it. True or False - The only ways that the partnership ordinarily can be dissolved are: by Vince's and Ari's agreeing to dissolve the partnership, or by the expiration of the partnership's term of duration. • c. The top-level management of a large corporation tends to be more representative of the entire population of workers and investors in Germany than in the United States. • d. In the course of forming the corporation, the promoter may incur costs, make contracts, and do other acts in furtherance of the corporation. • e. All corporations have one or more agents that act on the corporation's behalf.

Correct Answer b. Vince and Ari were in a partnership, but they now want to end it. True or False - The only ways that the partnership ordinarily can be dissolved are: by Vince's and Ari's agreeing to dissolve the partnership, or by the expiration of the partnership's term of duration. Answer: False; Barron's page 328. The death of a partner can also dissolve a partnership. a. While some federal laws regulate corporate activities, corporations are generally regulated at the state level. Answer: True; LSB 154, 161 ("Most corporate law is state law"); Barron's page 357. c. The top-level management of a large corporation tends to be more representative of the entire population of workers and investors in Germany than in the United States. Answer: True: LSB 168. The Supervisory board of large German Corporations is more representative of workers in terms of union and employee representatives appointing oversight on the management board (investors still also get to select other members of the management board as well). d. In the course of forming the corporation, the promoter may incur costs, make contracts, and do other acts in furtherance of the corporation. Answer: Corporations. True; LSB 162; Lectures; Barron's page 362 - "In the course of forming the corporation, the promoter may incur costs, make contracts, and do other acts [e.g., lines up employees and suppliers] in furtherance of the corporation." Because the corporate entity is not yet created, the promoter is potentially liable for his actions - not protected by a corporate shield. e. All corporations have one or more agents that act on the corporation's behalf. Answer: True. Discussed in lectures; Barron's pages 297 ("Since corporations can act only vicariously through employees, officers, and agents, corporations could not function at all and would cease to exist") & 315 ("A corporation has no "arms and legs" of its own. It can act only through agents").

. Which of the following scenarios best describes an advantage of forming a sole proprietorship? A) Elizabeth had been in the bee keeping business for many years and unfortunately passed away last week. The business she owned ceased to exist at this point. B) Pablo visited Rex's shop and slipped and fell, injuring his leg. Pablo sued Rex and Rex was held personally liable. C) Bernie was in dire need of a loan in order to keep his business running but every bank he went to was wary of extending him credit. D) Jennie owned a successful shirt company and made a lot of money this year. She was able to pocket all of her profits and did not have to share with anyone else. E) Maria's income from her business this year was taxed on two levels rather than one.

D. LSB 134-135, Barron's page 320: D) is the correct answer because an inherent advantage of starting a sole proprietorship is that the owner does not have to share in the profits other than paying taxes or sharing with spouse or dependents. The other examples describe disadvantages of a sole proprietorship, such as unlimited liability or it being an unstable business format.

. Choose the true statement: a. Two key advantages of a corporation are perpetual existence and the limited liability for owners in their role as owners. b. Corporations are 60% of all U.S. business entities. c. Generally, partnerships are subject to double tax because the partnership is taxed at the entity and partner level. d. Quincy wants to start a company so that he can sell energy drinks. True or False: Quincy should choose a corporation because a corporation is the only entity that provides limited liability for all investors. e. Shareholders within a company are ineligible to file suit due to the provisions in the Clayton Act, which explicitly deny such action.

a. Two key advantages of a corporation are perpetual existence and the limited liability for owners in their role as owners. Answer - True; LSB 148; Barron's page 308; Lectures. Shareholders have limited liability, and another advantage is perpetual existence. b. Corporations are 60% of all U.S. business entities. Answer: False; LSB 132 (corporations are only 20% of all entities). c. Generally, partnerships are subject to double tax because the partnership is taxed at the entity and partner level. Answer: False; LSB 132. The partnership itself does not pay taxes - taxes are paid by each partner at the individual level. d. Quincy wants to start a company so that he can sell energy drinks. True or False: Quincy should choose a corporation because a corporation is the only entity that provides limited liability for all investors. Answer: False. As discussed in lectures about limited partnerships, LLCs, and corporations, LLCs also provide limited liability for all investors. LSB 132-133. e. Shareholders within a company are ineligible to file suit due to the provisions in the Clayton Act, which explicitly deny such action. Answer: False. Lectures; LSB 166; Barron's page 390. Shareholders can sue the corporation or file a derivative action on behalf of the corporation.

. Zeb lends money as a business loan to Ashlee, a sole proprietor whose business name is Pants and Shirts. Who is liable to repay Zeb? a) Ashlee b) Pants and Shirts c) No one d) a and b e) the bank

A. Barron's page 320. One of the disadvantages of a sole proprietorship is that "personal and business assets are not separate," so Ashlee would be fully responsible in paying Zeb back. LSB 135.

. David is the incorporator of a new corporation named PASS. In the application, he wrote that the corporate name is PASS and that the corporation would focus its efforts on helping students succeed in business law themed courses. David checked the box that PASS would last forever. Lastly, David filled in addresses and information about the hierarchy. A few days later, David is alerted that his application was incomplete. Which of the following did David likely forget? a. Information about its Power Limitation. b. Information about its number of stocks and the classes of stock. c. Information about its Bylaws. d. Information about its Day-to-Day operations. e. Information about the transferability of its stock.

B. Barron's page 373 (Generally... statutes require information about the number of shares of stock of all classes that the corporation has the authority to issue, the number of shares of stock of each class, the par value, and other matters concerning both equity and capital.").

. Jim, Bob, and Sally have agreed to start a business. Jim will be in charge of the day-to-day activities, Bob is a passive investor, and Sally will be the business' attorney and accountant (she is very smart). They agree that they will have an equal vote to make key business decisions. Jim will own 40%, Bob will own 20%, and Sally will own 40%. They will share in both the profits and the losses of the business. They do not file any paperwork with any government agency before opening their business. What type of organization have they created? A. Corporation B. Limited Partnership C. General Partnership D. LLC E. No organization

C. Barron's page 330. This hypothetical goes through the three factors courts consider in determining if an implied agreement exists for a partnership (page 330); partnerships generally do not require a special formality beyond a partnership agreement. Joint ownership exists between J, B, & S; the three partners agree to share the losses and profits equally, and they have equal rights for making key decisions.

. Which of the following is the owner of an LLC? a. Its partners b. Its stockholders c. Its members d. Its president e. The government

C. LSB 145 ("the LLC permits investors to manage the business yet not be personally liable for the business debts"). Barron's page 321 - Item 5 — a LLC is owned by "the members." "Note that with the LLC, the owners are termed "members," not corporate 'shareholders' or partnership 'partners'." LSB 145.

. Johnny is an investment banker preparing for the merger of two large companies. Once the merger happens, Johnny is sued because one of the companies committed fraud. The plaintiff argues that Johnny did not do enough to prevent the fraud of the company. Is Johnny liable? a. Johnny is not liable. Johnny did his due diligence in using the information provided by the companies to conduct the merger. b. Johnny is not liable. Johnny was a victim of fraud as much as the plaintiff. c. Johnny is liable. It is his responsibility to investigate into the company further, and he neglected this responsibility. d. Johnny is liable. By not stopping the fraud, Johnny is as much liable as the company. e. Johnny is not liable. He was not acting on behalf of the fraudulent company, making him free of any liability.

A. "Due diligence ordinarily is not expected of investment bankers advising companies about mergers; that is because financial advisers typically do not look behind the numbers in audited financial statements, and — as a standard practice — the investment bankers (or other financial advisers) state in documents known as "fairness opinions" that they have relied on the merging companies to tell the truth (and thus have not conducted their own investigations into the numbers supplied by the companies)." LSB 162. While all of these other answers seem as if they could be right, A is the best answer. Investment bankers are expected to use the information they are provided.

. Jill was hired as a sales associate for Kates Cupcakes, an upscale dessert-and-drinks (smoothies, tea, coffee, and almost anything else imaginable). In her third week on the job, Kate, the owner, simply says to Jill, "Hey, Jill, we need a blender - when you get a chance in the next couple days, please buy a blender." Jill goes to Target, sees that there are five different models, all of which are newer and, in some ways, "better" than the older models Kates Cupcakes has been using. Deciding that in the long run it might be most cost-effective and more in keeping with the Kates Cupcakes image, ambiance, and products, Jill buys the most expensive blender that Target sells. When Kate finds out, she is furious and refuses to pay. Kate says that all she wanted was a simple blender. Who would win in court, Jill or Kate, and why? A. Jill; she was working as an agent for Kate, the principal. B. Kate; she did not tell Jill to buy the most expensive blender. C. Jill; she thought the store should have a top-of-the-line blender. D. Kate; she wanted a blender with the most value. E. There is not enough information to answer the question.

A. As observed at LSB 117-118, principals can be held liable for contracts into which their agent has entered. Additionally, Kate, if she wanted to get a specific blender, stay within a cost range, or provide other specifications, should have clarified her needs for the blender Jill was supposed to buy. Jill could certainly assume the high-end blender was the appropriate blender for a high-end place.

. Which of the following best describes a formation of a general partnership under the Uniform Partnership Act? a. Demi and Toni have a proprietary cosmetic idea that they wish to bring to market. The two come together as joint owners, agreeing to run the company together and to share in the profits and losses. b. Abby is seeking to bring on a partner to her baking company. She has yet to hire anyone else to join the business with her. c. Laertes and Osric decide to form a business to start selling their clothing subscriptions service. There is no business agreement, but Laertes has assumed all the profits and losses of the venture. d. Albert and Jimmy ran a successful Gators apparel business for many years. Albert unfortunately passed away, but the apparel business was still being operated by Jimmy after Albert's death. e. Andy and Raymond worked as contractors for a well-known construction company in town. They decided to branch out and start their own construction business, with Andy taking all the management rights of the company.

A. LSB 138-139; Barron's pages 319-325. Demi and Toni have formed a business with the essential characteristics of an association of two or more persons carrying on as co-owners for profit. Answer B is incorrect because Abby has yet to bring on another person to carry on as a co-owner of the business. Answer C is incorrect because a key element of a partnership is the sharing of profits and losses. Answer D is incorrect because a characteristic of a partnership is that it terminates after the death of a partner. Answer E is incorrect because management rights were taken over by Andy and the other purported partner did not have any decision rights in thee company.

. Charlie Sheen and Emilio Estevez run a novelty company, Winning, Inc. Because neither Charlie nor Emilio currently have any other source of income, they approach Martin Sheen for additional funding as they expand. Martin agrees to contribute money in exchange for a special share of the profits but does not want to manage or control the business or risk losing more than his investment. Martin could best be described as a(n): a. limited partner b. agent c. independent contractor d. partner by estoppel e. ex post facto partner

A. LSB 144 (Limited partners are investors, somewhat similar to corporate shareholders, not managers. "Limited partners contribute capital and receives a special share of the LP's profits." However, limited partners only risk what they put in. In return for risking only their investment, limited partners have no management or control of the business "and are not held personally liable for partnership obligations beyond the amount of his/her capital contribution."); Barron's pages 320-325.

. Warren Buffet is a new stockholder of Wells Fargo & Co, a well-known bank. He has been a stockholder for around seven months and has 4% ownership in the bank, Buffet believes one of the managers is using the company's money for his personal use. He suspects that this company manager bought his private jet with the company's money; so, Buffet writes a written demand to inspect the financial books. Would Buffet be able to gain access to this information or not? a. Yes, because he met all the requirements in order to review the financial books. b. Yes, because he wrote a written demand, had ownership of 4% of the company and had a proper purpose. c. Yes, because he wrote a written demand and proper cause. d. No, because he did not have enough ownership of the company. e. No, because he did not have a proper purpose.

A. LSB 165. The Correct answer is A because he met all three requirements. Warren has had the stock for longer than 6 months, had proper purpose to review the books and wrote a written demand.

. Yumi is a shareholder in a Public Corporation. Yumi notices that the corporation is losing revenue, causing shares to drop drastically. Yumi believes this is because of Corporate Negligence and poor business judgment. Yumi wants to take measures into her own hands. What can Yumi do? a. Sue the Corporation b. Manage the hiring and firing of corporate employees c. Appoint new Corporate Officers d. Amend the Bylaws e. A Shareholder cannot do any of these things

A. LSB 166-169. Shareholders can sue the corporation. A derivative suit is a lawsuit "brought by one or more shareholders on behalf of the corporation and for its benefit. Since these suits are not for the benefit of the suing party, any recovery belongs to the corporation." Barron's page 390. The shareholder's power to sue arises out of any owner's general power to oversee his/her business (his/her ownership interests). Here, Yumi is a shareholder and can sue the Corporation to get it back in line with her interest. Therefore, A is the correct answer. B-E are incorrect. B-D are things that the Board of Directors do.

. Tonya is the president of Big Corporation. Big Corporation is looking for land on which to build a new facility. Tonya locates suitable land but purchases it for herself with plans to sell the land at a profit at a later date. Rick, the majority shareholder of Big Corporation, hears about Tonya's purchase and complains to her about it. She tells Rick that she viewed and purchased the land on her own time and that she did not breach any duties owed to the corporation. Rick tells her that she should reconsider and that he plans to discuss the matter with the rest of the board. Did Tonya violate any duties owed to the corporation? A. Yes, by buying the land for herself without disclosure to the corporation, she violated the corporate opportunity doctrine. B. Yes, by buying the land for herself without disclosure to the corporation, she violated the duty of responsible decision making. C. Only if the land involved was worth over $50,000 did she violate any duties because any smaller amount would be considered a violation of the profits-expectancy test. D. No. E. Yes, assuming that Rick had a right of first refusal.

A. LSB 171. Directors or officers violate their fiduciary duty if they personally avail themselves of a business opportunity that should have been reserved for the corporation. In the instant case, Tonya violated the corporate opportunity doctrine by personally availing herself to a business opportunity that she should have reserved for the Big Corporation.

. Charles tragically passed away while snorkeling on vacation. He left behind a large multinational corporation, of which he was the CEO and a major stockholder (10% of all the common stock) and several hundred thousand dollars in cash, the source of that cash being entirely unclear - although it may be corporate property. Which of the following parties would not be required to act as a fiduciary in regard to the allotment of the discovered money? a. Shareholders of the corporation b. Directors within the corporation c. Officers within the corporation d. Professionals - e.g., lawyers and accountants - working for the corporation. e. A, B, and C.

A. Lectures; LSB 169, 174. Officers and directors have general fiduciary duties of honesty, fairness, and disclosure of information. Directors have duties to shareholders and the corporation. Officers have duties to the corporation. Barron's pages 357-360.

Juanita, Michael, and Sarah, founders of a worldwide corporation, wanted to change specific provisions in their corporation's charter. What process should they take in order to complete this action? a. The incorporators should execute a formal Articles of Incorporation through the state's corporate commission. b. The incorporators should file an application abiding by the rules under U.S. Statutes § 607.804. c. The shareholders should contact the State's corporate headquarters and ask for a corporate certificate. d. The board of directors should file the Articles of Incorporation with the Governor in the state in which the corporation sits. e. The shareholders should publicly record the number of shares held by each shareholder to the corporate commission where the corporation sits.

A. See LSB 163. "The charter (sometimes called the articles of incorporation, or articles) is the grant of corporate existence, the birth certificate of the corporation. This formal document, executed by the state through its corporate commission, is the source of corporate authority."

. Manny, Moe, and Jack open an automotive store together that becomes very successful. Moe and Jack only made financial contributions to the venture where Manny actually ran the store. A large part of the store's success is due to the excellent location. When their lease is up Manny decides to renew the lease, but only in his name. Which of the following is most accurate? A. Manny cannot sign the lease in his name only because he has a duty of loyalty to the venture. B. Manny cannot sign the lease only in his name because an individual who is a member of a partnership cannot sign an enforceable contract during the lifespan of the partnership. C. Manny can sign the lease because partnerships have a limited life span. D. Manny can sign the lease because Manny was the only one who worked in the store. E. Manny cannot sign the lease because Manny did not contribute money to the venture and only those partners who have made financial contributions to the venture have the authority to bind the partnership.

A. The question follows the Meinhard v. Salmon case. Manny cannot sign because he has a duty of loyalty to Moe and Jack. LSB 139; Barron's page 334. Answer B is incorrect because individuals can sign contracts while in a partnership (they just must disclose their signings as well as include the partners in the name of the contract). Answer E is incorrect because financial contributions are not the only way to be associated with a business. Manny ran all the operations of the business, thus making him a partner.

. Milo Inc. holds a substantial debt to creditors, and there has been verified commingling of personal and corporate assets. The use of shareholders' control of Milo Inc. for an improper purpose, harm to the plaintiff and the corporation being as an "alter ego of owners," have all been shown in this case. The court may apply and creditors collect from . A. Piercing the Corporate Veil; shareholders B. Sarbanes- Oxley; the board of directors and/or key corporate officers C. Piercing the Corporate Veil; corporate creditors D. Sarbanes-Oxley; shareholders E. Delaware corporate law; owners

A. To pierce the corporate veil, victims must prove: (1) The shareholders were acting on behalf of the corporation to commit fraud; and (2) The corporation was merely an alter ego 3) The plaintiffs suffered actual harm. LSB 176.

. Which U.S. state is considered the preeminent corporate authority due to extensive court precedence and a very corporate friendly Court of Chancery? A) Nevada B) Delaware C) New York D) California E) Connecticut

B) LSB 155-157: B) is the correct answer because Delaware provides state corporate law that is very favorable to many large businesses. There is freedom of management and a relative certainty in the law that makes it attractive for companies to incorporate in Delaware and be governed by its laws.

. Tom goes into business with Joe opening a line of pizzerias. Tom only invests money into the business, while Joe runs the day-to-day management. Tom only risks what he puts in, but Tom does not have any control over the business. What term best describes Tom's role? A. General Partner B. Limited Partner C. Agent D. Principal E. Shareholder

B. Barron's page 434; LSB 144. Limited partners only risk what they put in. In return for risking only their investment, limited partners, unlike general partners, have no management or control of the business "and are not held personally liable for partnership obligations beyond the amount of his/her capital contribution."

. Choose the situations that does NOT demonstrate an advantage of "The Rod Ripley sole proprietorship." a. Rod originally sold baked goods out of his home. Sales were so successful that he created a website "Sweet Delight" and sold his goods through the website. b. Rod was talking about expanding the business with his father, who managed the finances of Sweet Delight. The father, as was his right given his arrangement with Rod, refused to agree to expand. c. Rod grew tired of operating the business, so he retired and gave his son control of Sweet Delight. d. When filing his tax return, Rod was able to include Sweet Delight's income in his personal income tax and not have a separate tax liability. e. Rod decided to open up a mechanic shop, so he rented out a building, put up a sign, and got to work fixing cars.

B. LSB 134-135. B demonstrates a requirement of approval before expanding. If it was a sole proprietorship, Rod would not need anyone else's approval before making a decision. A demonstrates the advantage of total control over the business. C points out how easy it is to dissolve a sole proprietorship. D is correct in the tax advantages of a sole proprietorship. E is similar to A in how total control is helpful.

. Dwayne and Reggie were interested in starting a fitness club but needed additional capital in order to lease a storefront and purchase gym equipment. The two approach Ace Equipment's Inc. for funding. Ace agree to provide funds in exchange for a share of the company's profits, but did not want to manage or control the fitness club. What term would best describe Ace's role in the company? A) Corporate Director B) Limited Partner C) Professional Association D) General Partner E) De facto agent

B. LSB 144, Barron's 320-325: B) is the correct answer because a limited partner is typically an investor that is similar to a corporate shareholder but does not partake in any management of the company. A limited partner only risks what it puts in, and it is not generally held liable for partnership obligations.

. Every Saturday night, Maggie and a group of friends go bowling as part of their activities in Baystate Bowling League LP ("Baystate"), a group of bowling aficionados that have formed a limited partnership dedicated to reviving the sport. Baystate sells T- Shirts, shoes, balls, and other items as well as the services of bowling coaches who instruct paying customers in how to be better bowlers. This Saturday, Maggie, a general partner in Baystate, calls ahead and makes a contract with Bob's Bowling Alley Company ("Bob's Co.") to reserve the use of two bowling lanes for Baystate. Does Bob's Co. have a legal contract with Baystate to pay for the reservation and fees? a. No, Bob's Co. only has a contract with Maggie because the group of friends is not organized in a legal club or business. b. Yes, Bob's Co. has a legal contract with Baystate. Regardless of whether Maggie and her friends actually get together every Saturday night to go bowling, Bob's Co. can consider Maggie to be an agent who can bind Baystate. c. Yes, because even though Maggie called, multiple people will be bowling. d. No, in these circumstances, Bob's does not have a legal contract with anyone. e. Yes, assuming that Baystate has at least one shareholder who is a citizen of the state in which the Bob's Co. bowling alley is located.

B. LSB 144-145. Maggie is a general partner of Baystate and is certainly acting within the scope of her authority as a general partner.

. Lacey, a Florida resident, just established her new business, Colorful Creations, Inc., in Delaware. Her business operates in the state of Florida and manufactures unique home décor and furniture. So far, Lacey is pleased to see that she is making a positive return on her investment. The income seems promising for continued growth, which is excellent news considering Lacey has filed a form with the IRS that should ensure only Lacey, not her business, owes taxes on Colorful Creations, Inc. income. Which of the following best describes Colorful Creations, Inc.'s business form? A. Public, domestic, S Corporation B. Private, foreign, S Corporation C. Professional, non-profit, C Corporation D. Public, Close, S Corporation E. Private, domestic, C Corporation

B. LSB 157-160. The "Inc." designation in the name of the business indicates that it is a corporation. Lacey's business does not serve a governmental or public purpose; therefore, it is not a public corporation. Because this corporation does not have the typical protections an ordinary corporation does (i.e., Lacey is liable for the losses and taxes), Colorful Creations, Inc. is an S Corporation. It is also a foreign corporation because it operates in a state (Florida) other than the state it is incorporated in (Delaware).

. Shareholder primacy, a traditional corporate governance theory, places shareholders' interests above others. Which scenario best describes this in practice? A) Landstart Inc. is a major trucking company in the southeast and has many contracts with large manufacturers. Landstart's contracts are negotiated in good faith and offer the manufacturers discounts for bulk shipping. B) The board of directors of Racket Co. must make a big decision regarding fossil fuels. The company's next project would benefit the owners of the company but comes at the expense of the surrounding town. C) Carol has been working for Lightpost Inc. for over thirty years. She receives great benefits as an employee, even though she has no ownership stake in the corporation. D) Customers of Flavor Co. are loyal to the brand and always feel welcome when visiting storefronts. Flavor Co. focuses on the consumer experience, even if it means greater expenses. E) Frontier Corp. is a home goods corporation that sells goods from various suppliers. These suppliers enjoy doing business with Frontier due to its communication and favorable contract terms.

B. LSB 160-161; Barron's pages 365-366. This is the correct answer because it embodies the shareholder primacy model and focuses on the best interests of the shareholders at the expense of others. A) is incorrect because these manufacturers are considered stakeholders and Landstart considers the manufacturers interests in the distributing. C) is incorrect because this shows a corporation considering the employees that do not have a stake in the corporation. D) is incorrect because it shows the corporation taking a cut in gross profits by considering the needs of customers, who are stakeholders in the corporation. E) is incorrect because the corporation is addressing non-financial interests of suppliers in negotiation fair contracts.

Tessa scheduled a meeting with a potential entrepreneur who wishes to invest in a corporation. Tessa is on the board of directors for Buffa Bortolotti Corporation, and she also serves as a key employee. While talking with the entrepreneur, Tessa realizes that the entrepreneur would be a great source of capital for her sister's new florist business. Tessa suggests the entrepreneur invest in her sister's new business. After the meeting, Tessa's sister gets a $2,000,000 investment from the entrepreneur. Buffa Bortolotti gets nothing. Tessa violated the: a.Business Judgment Rule b.The Corporate Opportunity Doctrine c.Federal Antitrust Regulations d.Americans with Disabilities Act e.OECD, WTO and/or FTC guidelines.

B. LSB 171-173; see LSB 171 (''Officers, directors, or high-level employees must take opportunities first to the corporation before pursuing these opportunities for their own personal benefit (or to benefit others aligned with them - e.g., a family member, a friend, a personal creditor).") Answer A: The Business Judgement Rule describes an officer making an informed business decision, which very likely relieves the officer from court questioning of their judgment, no matter the outcome. This does not apply because Tessa never made a decision on behalf of the company. The most relevant information is that Tessa failed to disclose an opportunity to the corporation first. Answer C: Antitrust regulations deal with fairness and competition in a certain industry, mostly to avoid formations of monopolies. This is not relevant to the situation. Answer D: The ADA would very likely be relevant if some form of discrimination against those with disabilities was present. That is not the case here. Answer E: The OECD, WTO, and FTC guidelines concern fair trade, both internationally and domestically.

. Teldar Paper Co. entered into a contract with Anacott Steel Corp. for the sale of its mineral holdings. The transaction is beyond Teldar's powers. Which of the following parties, for the reason stated, may properly assert the lack of powers in order to challenge and try to stop or undo the sale? A. Teldar Paper Co. B. A shareholder of Teldar Paper Co. C. Anacott Steel Corp. D. Anacott Steel Corp., but only to change the terms of the contract (not stop or undo the sale). E. Teldar Paper Co., but only to change the terms of the contract (not stop or undo the sale).

B. LSB 178 about ultra vires. Only two parties can obtain an injunction against corporate activity: (1) shareholders and (2) the state

. Lisa is a shareholder of a corporation. She recently learned that her brother, one of the corporation's officers, is not following the corporation's procedures. Can Lisa and her fellow shareholders obtain any relief? a. Yes, shareholders can take an action against Lisa's brothers if he happens to be a director, also. b. Yes, shareholders can obtain an injunction against the company's doing what Lisa's brother authorized in violation of corporate procedures. c. Yes, shareholders may obtain compensation, but they cannot get any orders to undo/enjoin what Lisa's brother did. d. No, shareholders cannot sue their own company e. No, as long as the board of directors, within the state's statutory framework (e.g., two years after an unauthorized action or one year after a complaint to the board, whichever is sooner), satisfies the action by Lisa's brother.

B. LSB 178. This is an example of ultra vires. It is usually used when officers fail to follow the set corporate procedures. It allows shareholders to gain relief against the officers through injunctions and compensation. Note that if Lisa, and perhaps other shareholders, sue on behalf of the corporation, that would constitute a derivative suit.

. Ned, Catelyn, and Peter decide to start a business. Peter solely wishes to serve as a passive investor. In exchange, Ned and Catelyn will split the losses amongst themselves and Peter will take less than a third of the profits. Ned and Catelyn agree. They file the appropriate paperwork with the State. After a year of profitability, the business begins to fail. Ned and Catelyn quickly learn that Peter opened up a business that was directly competing with theirs. Ned and Catelyn file a claim against Peter. The claim will: a. Succeed because Peter breached his fiduciary duty toward the business. b. Succeed because Peter did not inform the two prior to competing. c. Fail because Peter owed no duty to the business. d. Fail because shareholders do not owe each other a duty. e. Fail because Peter is not a partner of the business.

C. Barron's page 322; LSB 144. The three created a partnership because they are splitting profits and losses, which is not allowed for corporations. Peter is a limited partner because he is working as a passive investor and receives a portion of the profits. The required paperwork is filed. The allocation does not eliminate his status. Limited partners are not agents or principals of the business and therefore they do not owe a duty to the business. Therefore, Peter has breached no duty to Ned and Caitlyn. Peter may have behaved unethically, but not illegally.

. States have chosen to authorize the limited liability company because: a. LLC ownership is more readily transferrable than corporate ownership b. LLCs do not pay a corporate income tax on earnings, nor is income taxed to the shareholders. c. Depending on the issue, LLCs are treated like either a partnership or like a corporation. d. LLC operations are more transparent than corporations or partnerships and thereby offer more possibilities for socially beneficial business enterprises e. LLC law throughout the states is fairly certain.

C. Barron's pages 365-371, Lectures, LSB 145. More flexible than general or limited partnerships, the LLC permits investors to manage the business (comparable to a partnership) yet not be personally liable for the business debts (comparable to a corporation). Page 368 of the Barron's book states, "A 1988 IRS ruling that, for federal tax purposes, LLCs are to be treated like partnerships has led every state to authorize the LLC."

. On his own and without any particular filing with the state, Alex opens up his own pizza shop. Alex is incredibly successful, and he pockets all the shop's profits. One day, Margaret gets sick from the pizza and sues Alex. What kind of business is Alex's pizza shop? A. Partnership B. Corporation C. Sole Proprietorship D. Limited Liability Corporation E. Franchise

C. LSB 132. In the question, it mentions that Alex opens up his own pizza shop. The lack of a filing means it is not a corporation or an LLC. There is no other owner, apparently, so it is not a partnership. That Alex was on his own means, there was no partnership or franchise. He is being sued personally, and he is a sole proprietorship, as is evident at LSB 132.

. Mike constantly sees Ben and Jerry working at a coffee store. Mike strikes up a conversation because he is interested in purchasing the property. He is not sure with whom he should negotiate. Mike has a great relationship with Ben, but Mike does not get along with Jerry. Mike did learn from their conversations that Ben owns the property and does not share profits or losses with Jerry or anyone else. Neither Ben nor Jerry has filed any paperwork to operate the business. Jerry manages the day-to-day operations. With whom must Mike negotiate? a. Jerry, because he controls the ordering of supplies b. Jerry, because he controls who to hire and fire c. Ben, because he owns the property as well as receives the business profits and/or is responsible for its losses d. Jerry, because Ben did not file any paperwork e. Ben and Jerry, because they are both "higher ups" in the store.

C. LSB 134. The store is a sole proprietorship and since Ben is the only one who receives the store profits, he is the sole proprietor. Even though he did not file anything, this does not invalidate his business. Sole proprietorships do not require filings.

. Rusty and Betsy are partners running the general partnership, Ice Cube Enterprises, where they make and sell ice cubes. On behalf of the partnership, Betsy renews the lease on the building where they house the ice cube machines without telling Rusty. Rusty and Betsy fall on hard times, and Ice Cube Enterprises goes bankrupt. Rusty is: a. Personally liable for only the debts of the company excluding the lease. b. Personally liable for exactly half the cost of the lease, if Betsy cannot pay her half then too bad for the lessor (the landlord). c. Personally liable because Betsy was acting within the scope of the partnership authority. d. Not personally liable because General Partnerships are protected business ventures. e. Not personally liable, Betsy should have disclosed the renewal of the lease before it was made.

C. LSB 138 (Partnerships have mutual agency - liability risks for all partners (each partner is an agent) - a partner had better trust his partners; it is a great problem if business (or marital!) partners are incompetent or, even worse, dishonest - respondeat superior (each partner is bound by the acts of the other partners, insofar as those acts are within the scope of partnership authority).). Barron's page 302. Rusty may have a claim against Betsy for her not keeping him apprised of her activities, but - as Betsy had express, implied, and/or apparent authority to do what she did, Rusty does have liability with respect to the partnership's obligations she incurred on the partnership's behalf. The Barron's text mentions that partners are principals for each other and their employees. Because of this and due to the doctrine of respondeat superior (discussed at LSB 116), the partners are all civilly liable for breaches of trust and/or torts committed by just one partner (or even an employee) so long as that person is acting within the scope of his/her authority.

. Timmy decided to open up a lemonade stand. Timmy's sister, Monica, provided Timmy with the table, lemons, water, ice, and cups. She tells him how long he can sell and then heads inside the house they share. Unfortunately, business is slow. Therefore, Timmy decides to make a sign. The sign reads "Lemonade, Inc." Timmy finally receives a customer, Carly. She asks Timmy if she could use Timmy's logo on her own lemonade stands. Based ONLY on the facts above, which of the following best describes Timmy's form of business? a. It is a limited partnership because Monica had no control. b. It is a sole proprietorship because Timmy makes all decisions regarding the stand. c. It is a general partnership because Monica invested and made managerial decisions. d. It is a corporation because Timmy provided notice of incorporation through the sign. e. It is a franchise because Carly wants to use Timmy's trademarks.

C. LSB 138. A partnership may be entirely run by one of the partners, while the other partner provides capital or supplies for the business to operate. Timmy makes the decisions, but Monica is a partner because she provided the capital and supplies of the business. D is incorrect, a business must be registered to become a corporation. Notice alone is not sufficient. E is incorrect because a franchise is not a form of business. A is incorrect because Monica designated the "store hours."

. Sam invested $10,000 in a limited partnership with Michael and Fiona. Sam was a general partner in the limited partnership. The partnership failed to pay Jesse $30,000 for services on behalf of the partnership. Which of the following statements is generally correct regarding Sam's liability? A. Sam is liable for $20,000 because this was a limited partnership B. Sam is liable for nothing because this was a limited partnership C. Sam is liable for $30,000 because Sam was a general partner D. Sam is liable for $10,000 because this was a limited partnership E. Sam is liable for $10,000 or whatever remains on the debt (if a lesser amount) only if and when the partnership dissolves.

C. LSB 143-144. A general partner in a limited partnership has "unlimited legal responsibility." If Sam were a limited partner, he would be responsible only up to the amount that he put into the partnership.

. Penny and Rachel want to be successful in life, so they decide to start a Limited Liability Company (LLC.) called Dream On. Dream On is found on making young kids become successful by giving them big goals to achieve through their lifetime. Penny and Rachel are thinking of changing Dream On into a General Partnership. Which of the following is not a benefit for Penny and Rachel to keeping Dream On as a Limited Liability Company? a. Limited Liability Companies are a hybrid between Partnership and Corporations b. Limited Liability Companies avoid double taxation c. Limited Liability Companies are easier to form and do not require filing with the State to be formed d. Limited Liability Companies have limited liability to their members e. Limited Liability Companies are more flexible and do not have board of directors

C. LSB 145. LLCs must file articles of organization with the state. Therefore, C is an incorrect statement, making it the correct answer. All of the other statements are true about Limited Liability Companies.

. In a particular European country, the corporation's purpose is not simply to maximize per-share-values like it is for the most part in the Unites States. Workers' councils are given extensive power in running companies and are able to contribute in many ways, including improving working conditions. In which European country is this mainly emphasized, and what is the name of this system? a. This system is mainly seen in Italy and its proper name is supervisory boards b. This system is mainly seen in France and its proper name is codetermination c. This system is mainly seen in Germany and its proper name is codetermination d. This system is mainly seen in France and its proper name is supervisory boards e. This system is mainly seen in United Kingdom and its proper name is supervisory boards

C. LSB 168 ("Under the German system known as co- determination, workers' councils are legally defined bodies that appoint half the members of a company's supervisory board"). The system's name is Codetermination, which is used in Germany.

. Jake owns 300 shares of State Ranch. Jake is not an officer or a director of State Ranch. Jake hears about a new opportunity with State Ranch's competitor from his brother. Jake acts on the knowledge and buys the stock of State Ranch's competitor and makes a lot of money. Is Jake liable for acting on the tip? A. Yes, because he acted on insider knowledge. B. Yes, because he is considered a tippee. C. No, because Jake is only a shareholder. D. No, because Jake owns less than 500 shares of State Ranch. E. No, because Jake is not considered a tippee.

C. LSB 171. Officers, directors, or high-level employees must take opportunities first to the corporation before pursuing these opportunities for their own personal benefit (or to benefit others aligned with them - e.g., a family member, a friend, a personal creditor). Shareholders do NOT have that duty. For example, you can own 500 shares of Microsoft while owning 500 shares of Apple; but a director or officer of Microsoft ordinarily is not permitted to also be a director or officer of a competing company, such as Apple.

. A shareholder of Behemoth Corp. has sued the Behemoth Board of Directors. That Board failed to verify independently (audit) certain corporate financial statements. Those statements turned out to be false. In reliance on the statements, the Board committed large sums of money - borrowed at steep interest rates - in order to expand. Once modified financial statements were finally released, showing that Behemoth's short-term prospects were not very rosy, Behemoth's stock price plummeted. Choose the true statement: A. the Board can avoid liability if it shows that it lacked scienter B. the Board is strictly liable to the shareholder and/or to the corporation itself C. the Board may avoid liability if it acted in good faith and in a reasonable manner D. the Board is liable because any negligence by its in-house accountants is automatically imputed to the Board E. all of these

C. LSB 174-175 (''In order to use the BJR as a defense, directors must act in good faith and with reasonable belief that their conduct legally and legitimately assists in achieving the corporation's purposes.").

Quinn is the sole owner and shareholder of Trucks & More. Brena was browsing the selection of 2021 Ford trucks when Truck's & More's employee hits Brena with a truck. Brena is in the hospital for five days. Brena receives a $200,000 judgment against Trucks & More. Brena learns that none of the vehicles on the lot are owned by Trucks & More and that the business is insolvent. She also learns that when people buy vehicles from the lot all the money is transferred to Quinn and not the business. Also, all the vehicles are being sold as new, but when new vehicles come in that Quinn likes he will drive them until a new vehicle comes he likes and reset the odometer. Brena wants to collect her judgment from Quinn. Will she be successful? A. No, because shareholders are never liable for the debts of the company. B. No, because it has not been shown that Brena tried to collect from Trucks & More. C. Yes, because Brena can pierce the corporate veil. D. Yes, because Trucks & More cannot pay the judgment. E. Yes, because Quinn's employee hit Brena.

C. LSB 176. To pierce the corporate veil, three elements usually must be proven: 1. shareholders' control of the corporation for an improper purpose; 2. the corporation's use as its owners' "alter ego"; and 3. harm to the plaintiff. Here, there was harm to Brena because she was injured. Trucks & More is essentially Quinn's alter ego because all the money from the sale of the vehicles goes to him, and he uses the vehicles. Quinn's control is for an improper purpose of using the vehicles and changing the odometers after to sell the vehicles as new cars. Answer A is simply wrong for at least two reasons: (1) many shareholders are also employees, officers, and/or otherwise serving in another capacity and thus may be held accountable; and, more importantly, (2) piercing of the corporate veil is possible (in order to have shareholders pay the corporate debt). Answers B and D are not requirements. Answer E is an element of the case against Quinn, regardless of whether the employee was directly hired by Quinn (not what is stated in the fact pattern) or is an employee of Trucks & More, as in fact is stated in the question. However, E is not, by itself, sufficient to establish Quinn's liability. C builds upon all the facts in the case, including that the employee hit Brena, in order to establish liability of Trucks & More. And then from there one goes to the vital issue, that the facts satisfy: the ability to pierce the corporate veil and hold Quinn liable.

. Albert and Alberta come up with an idea to start a business renting out boating equipment at Lake Wauburg. They do not formally sign a contract to enter into a partnership or file any paper work with the State, but each buys the resources needed to start the business. Albert buys 60 percent of the materials needed, and Alberta buys 40 percent. If the business does not have the money to pay all its creditors: a. Albert is the only one personally at risk because he, at least technically, owns more of the company. b. Alberta is the only one personally at risk because she did not invest as much in the beginning. c. Albert and Alberta are both personally at risk because they have entered into a partnership. d. Neither Albert nor Alberta is at risk because they can both just walk away from the business without personally losing anything. e. Alberta would be personally at risk for 60% of the debts, with Albert personally at risk for 40% of the debts (Alberta thus "making up" for the fact that she only contributed 40% of the materials to start the business).

C. Lectures; LSB 138 (and footnote); Barron's pages 302-310. Partners equally share in both the risk of losses and the potential for gains. "Simply agreeing to do something as co-owners, could result in a partnership (so partnerships arise expressly or implicitly)." LSB 140. Both Answers A and Answer B are incorrect because the amount invested is not important, as both expressed the intent to form a partnership. Answer D would be true if the company were structured under an LLC or Corporation. Answer E is incorrect because both partners share debts. The debts are not allocated based on an inverse proportion of the initial investment amounts.

. Ryan, Kyla, and Jeremy are all general partners in a business. Ryan suddenly and unexpectedly passes away. Assuming there is no partnership provision covering these matters, what happens, under traditional rules, to the partnership? a. The Partnership automatically continues on with just Kyla and Jeremy b. The Partnership automatically continues on with Kyla, Jeremy, and Ryan's estate all as equal partners. c. The Partnership automatically terminates and Kyla and Jeremy must dissolve the business. d. The Partnership automatically stops until Ryan's interest is sold at auction with half of the proceeds being given to Ryan's estate and the other half to the partnership. e. The Partnership automatically terminates; however, the Partnership automatically reforms itself into an LLC.

C. Lectures; LSB 141-143. A characteristic of a general partnership is that it may have a limited lifespan - i.e., the death of one partner can mean the death of partnership that is the traditional rule. The question itself refers to the traditional rule. Yes, the surviving partners get the dead partner's "power," but under the traditional rule the partnership is dissolved, and the survivors would reconstitute a new partnership in order to carry on new business.

. Tim and Brandon decide to rent a warehouse and divide up the space, which will then be leased separately to football teams so they can store their equipment. They both signed the contract, which was a 15-year lease. Tim had the good credit and the funding to pay for the warehouse, but Brandon had the experience needed to run the place. 15 years go by, and their business decision was a great idea as they both made a very big profit in the end. However, now that Brandon has more money, he decides to renegotiate the lease on the warehouse and run the business himself. Which of the following is most correct based on the above situation? a. Once a building lease has expired there is no obligation that belongs to either partner that says the partner must let the other person in on the decision. b. Case law in New York and other leading state courts has determined that the duty of partnerships generally ceases to exist simultaneously to when their agreement has expired. Based on this, Tim would likely have no case against Brandon. c. A highly influential 1928 New York case extended the duty of parties to a joint venture (and, by extension, to partners in a partnership) beyond what a contact states, and, based on loyalty, the opportunity to renew belonged to both Tim and Brandon. d. Jo v. Jill determined that one's obligation to a partner, based on loyalty, extends past the term of the contractual agreement, which formed the partnership. e. The Revised Union Partnership Act protects partners from breach of loyalty as exhibited in this example; leaving Tim with a case against Brandon.

C. This example is based on the court case Meinhard v. Salmon as presented in LSB 139; Barron's page 334. This case extended the duties of a partnership far beyond what it states, holding in the example that Tim should have had the option to renew the lease with Brandon. Jo v. Jill is the made-up case based on this example. Moreover, the RUPA does not touch on the subject of what happens after the partnership.

. Four months ago, Maria Shriver bought 4.5% of the shares of Arnold Schwarzenegger's company, Governator Co. Today, she wrote a letter demanding to see Governator Co.'s corporate books. Maria, Arnold's ex-wife, wants to find out if Arnold is secretly using the company to make payments to his illegitimate children. Her request will most likely... a. be denied because she must own at least 10% of the shares in order to see the books b. be denied because she does not have a proper purpose to see the books c. be denied because she has not owned the stock for more than six months d. be granted because she has owned enough stock for long enough and her reason for wanting to see the books is not relevant e. granted because any stockholder can view the books at any time; it is one of the privileges of being a stockholder

C. To inspect the books, a shareholder must meet these 3 requirements: (1) Ownership: 5% (or more) of the stock or at least 1 share for more than six months, (2) Make a written demand, and (3) Must have a proper purpose - to check one's investment, see how directors and officers are doing their job. (NOT to demand access to get secrets, or to dig up dirt, or perhaps to press political points). Note that even if Maria is partly motivated by anger personally toward Arnold due to Arnold's philandering, the request to find out information related to company payments still relates to the proper use, or not, of company funds - a proper purpose (so answer B is false). There must be 5% ownership or minimum of 1 share for 6 months (not present in this question- 4 months and does not own 5%), make a written demand (present), and must have proper purpose (could be present if paying his illegitimate satisfies how directors and officers do their jobs). Therefore, we can eliminate answers A, D, and E. LSB 165-166.

. Jesse, after taking a business law class, forms a chemical manufacturing corporation, Chem Co. (Chem). He is the sole shareholder. He does not keep records of any dividends and very little records of the corporation's accounts. One day, one of Chem's products injures a user and Chem is sued. Which of the following ordinarily is a reason for a court to hold Jesse personally liable? a. The corporation did not elect any directors b. If Jesse is not held liable, creditors would not be fully compensated. c. Jesse served as the CEO, CFO (Chief Financial Officer), and CTO (Chief Technology Officer) of the company. d. Chem's headquarters was at the same address as Jesse's lemonade business. e. Jesse decides to use some of the profits in the corporation - paid to him as a dividend - to pay off his personal debts.

C.For Answer C, please note that in a role as CEO, CFO, or CTO, the person is more likely to have carried out acts binding the corporation, and thereby ALSO being held personally liable. Agents committing a tort, while acting on behalf of the principal, can also be held personally liable. It is not Answer A: Many small corporations are close corporations, for which a board of directors is not required. It is not Answer B: The "piercing of the corporate veil is not to be granted simply because corporate creditors would otherwise not be fully compensated. There must be more proof." (quoting from Barron's page 360). See also LSB 176-177. Answers D and E are quite unlikely to be decisive in and of themselves. See Barron's pages 360 & 361; LSB 176-177. For D, just having two corporations at the same address does not mean that the owners/managers, if they are the same, must be personally liable. For E, the only problem would be if the dividend were not properly issued; nothing indicates that is the case. If issued properly, then the dividend is truly the recipient's property to be spent any way that the recipient chooses.

. Which statement about corporations is true? a. Corporations are a modern legal creation, originating during the industrial revolution of the 1800s. b. Corporate shareholders cannot serve on the board of directors. c. Corporations, when created, must choose between its state of incorporation and the state where it has its principal place of business as its place of domicile. d. Corporations are not protected by the Freedom of Speech Clause of the First Amendment because they are not persons. e. Corporations are subject to double taxation.

Correct Answer E; LSB 146--150. Corporations are subject to double taxation. A) Corporations existed before the 1800's. "In the early 1600s, for example, the Virginia colony's royally chartered company (the Virginia Company) authorized the new colony's first general assembly." LSB 155. Answer B - "Directors often are, but do not have to be, shareholders: "The board of directors is chosen by the shareholders to manage the corporation." LSB 168. Answer C - "A corporation's domicile, at least for federal diversity jurisdiction, is the state in which it is incorporated and also in its principal place of business (so the corporation is domiciled in one or two states)." LSB 158. Answer D - " Corporations are "persons" in the eyes of the law, they have many of the same rights that an individual does."

. Amanda and Alena own a general partnership named "The Theatre Boutique," which sells theatre props to theatrical professors. Alena notices that the lease on the building of "The Theatre Boutique" is running out and renews the lease solely in her name, signing it, and declaring the partnership void. Amanda: a. Should negotiate a new leasing payment to Alena on behalf of their partnership to keep the company's partnership running in the same building. b. Still owns at least half of the assets of the old partnership but must move the partnership to a new location. c. Moves into sole ownership of the assets but must move them out of Alena's building. d. Should take Alena to court for violating her fiduciary duty. e. None of the Above.

D. LSB 138-139 (Meinhard v. Salmon); Barron's page 334. Both partners in a general partnership have common ownership for any assets for the business. The New York Court of Appeals (the highest court in New York) found in Meinhard v. Salmon that not disclosing business opportunities to the other partner is a breach of one's fiduciary duty. As such, Answers A and B are incorrect because they imply that the action was legal. Answer C is incorrect because the partnership cannot be dissolved this way. LSB 138-140.

. Steve and Tony form a crime fighting business. During one of their jobs, they caused a lot of damage. They are now worried that the town they damaged will sue them to recover. There is not a lot of value in the business, so the city will likely go after Steve's and Tony's personal assets. Pick the correct conclusion: True or False: Since they organized as a partnership, the city will recover from Steve's and Tony's personal assets. a. True, because it is a sole proprietorship. b. False, because partnerships require creditors to pierce the veil c. False, because it is difficult to raise capital d. True because partnerships have unlimited liability e. None of the above

D. LSB 140. Partnerships have unlimited liability, which means a credit must go through the partnerships assets. If that is not enough, creditors can go after the personal assets of each partner. With this in mind, Answer A would be incorrect because it is stated in the question that Steve and Tony both form the company, which would not make it a sole proprietorship, but a partnership.

. Wilfred is a very successful banker and has a very good reputation. Simbola goes around to businesses and tells each business that he is partners with Wilfred even though he really is not. This gives businesses confidence to allow Simbola to pay them back in the course of business since he is linked to the good reputation of Wilfred. If Wilfred knows that Simbola is doing this and does nothing about it, then what type of partnership is formed to make Wilfred liable for money owed to the businesses? a. Limited b. Express Agreement c. General d. Estoppel e. Wilfred is not liable. The businesses should have done more research.

D. LSB 142; Barron's page 331; discussed in lectures. Partnership by estoppel is similar to the concept of apparent authority (agency by estoppel) found in agency law. The key question is: Did partners create or permit third parties' reasonable belief and reliance on a partnership [the belief that a person was acting as a partner-agent on behalf of his/her supposed co-owners/partners (the supposed partnership)]? If the answer is "yes," the third party may, in that context, be able to treat the business like a partnership even if it was not an actual partnership (e.g., the so-called partner was actually not a partner and thus not actually able to bind his/her "partners").

. Ashley is looking to start a service business in Gainesville. When deciding how to organize her business, Ashley considers the following: (1) She wants to create a separation between herself and the business entity; (2) She wants to avoid double taxation; and (3) She want to permit other investors to manage the business without facing personal liability for debts. Which of the following business structures would best meet Ashley's wishes? A. Corporation B. General Partnership C. Sole Proprietorship D. Limited Liability Company E. Limited Partnership

D. LSB 145. LLCs enjoy the advantages of avoiding double taxation, separation between owners and business, and a very flexible structure, with investors able to join in without worrying about being personally liable. With her first consideration being a separation between herself and the business, both Sole Proprietorships and General Partnerships do not create a distinct entity in the business, making the owners personally liable for debts (making Answers B and C incorrect). Her second consideration would make a corporation not suitable because a corporate structure leaves the possibility for double taxation; the only exception to this would be an additional, necessary feature not mentioned in the scenario, such as choosing the S corporation status) (So Answer A incorrect). Her last consideration makes a Limited Partnership unreasonable because limited partners are simply investors in the business that do not manage the business (if they did, they would be deemed general partners, which makes them liable for company debts, making Answer E incorrect).

. Consider the facts from the previous question on the lemonade stand. Now add this: Timmy's lemonade stand has become a massive success. Monica hires an attorney to create a corporation, LT's Lemons (LT), and Monica is the majority shareholder of the company, which receives a corporate charter from the state. LT opens lemonade stands all across the country. Hugo, a customer, purchases some bad lemonade and becomes ill. Hugo sues LT, and the verdict is more than all of LT's assets. Monica may be personally liable for Hugo's damages. a. True, because once a corporation's assets are depleted, shareholders are liable. b. True, because the corporation began as a partnership and is without limited liability. c. True, because Monica is the principal of LT and LT is considered an employee. d. False, because LT and Monica's finances are separate. e. False, because Timmy remains the founder of the lemonade stand idea. Timmy would be liable.

D. LSB 148-149. While Monica is a shareholder and founder of LT, she is protected by limited liability. Thus, any suits against LT would not affect Monica's finances. The only exception is "piercing the corporate veil", but it does not apply in this situation. Therefore, any excess liability will not fall on Monica or Timmy. C is incorrect because LT is a separate legal entity.

. Which of the following reasons could explain why a business might choose to organize as a corporation rather than as a sole proprietorship or general partnership? a. Corporations are less expensive to form b. Corporations generally face lower taxes c. Corporations face fewer regulations d. Corporations generally find it easier to raise capital e. Corporations are not subject to double taxation

D. LSB 149. D is correct according to the second basic corporate advantage listed in the textbook: "With corporations, it is usually easier to raise capital." Corporations are not as advantageous for a business owner as are partnerships with respect to these factors, found in Answers A, B, C, and E. Answer A - An advantage of partnerships is that they are inexpensive to form (Barron's page 326 - noting that partnerships usually have less organizational costs than do corporations). Answer B - Corporations may face the "double taxation" problem (LSB 149) and partnerships do not (Barron's page 326). Answer C - "The rigidity of the corporate form makes it easier to regulate and therefore provides more rent-seeking opportunities for politicians and interest groups than if parties were left to choose their business form." LSB 150. Answer E - Typical C-Corporations are faced with double taxation, whereas sole proprietorships and partnerships do not. LSB 149.

. After a messy and costly divorce from his wife, Captain Jack Sparrow needs a diversion and decides to go into business with his cousin, One-Eyed Jones, selling swords, eye patches, and other pirate paraphernalia, as well as, of course, rum. The business is managed by other family members, who, amongst themselves, hold all the stock of the company. After a year, however, management decides it must dissolve the business because it has experienced severe losses, mainly due to Jack's consumption of the rum. In order to do this, a court resolution is required. Jack and his family are engaged in which business organization? a. partnership b. professional corporation c. limited partnership d. close corporation e. cooperative

D. LSB 158; Barron's pages 363-367. In close corporations, stock is held by a limited number of persons in management and run like a partnership. Close corporation statutes are intended to combine the management flexibility of a partnership with the liability protection of a corporation.

Choose the true statement: A. Only shareholders may be directors. B. In about 30 to 35 states (roughly, two-thirds of the states), the incorporation statutes permit a corporation to have perpetual existence. C. Bylaws are provisions for a corporation that are more detailed than the charter. In their detail the bylaws can go into greater length and even contradict the charter. D. Boards of directors typically have responsibility for decisions involving whether to authorize issuance of additional stock. E. Partnerships are called, for tax purposes, S corporations.

D. LSB 169. Directors' powers: BOARDSCAM. Answer A - "Directors often are, but do not have to be, shareholders: "The board of directors is chosen by the shareholders to manage the corporation." Board of directors are typically shareholders, but the position is not ONLY limited to them. LSB 168. Answer B - "Most modern corporations are usually granted perpetual existence as a routine charter provision, although corporation existence may be for a stated or limited period of time." LSB 153. Answer C - "Bylaws: Amendments (including Date and Purpose), Fiscal Year adopted by the corporation." LSB 164. Answer E- "For tax purposes, S corporations are treated like general partnerships." Partnerships are not called S corporations, S corporations are just treated as a partnership. LSB 159.

Choose the false statement. a. Statement: The business judgment rule, if it is to be invoked, is a defense against suits claiming corporate directors and/or officers should be held liable for an allegedly poor decision. b. Statement: Partnership by estoppel is a form of apparent authority, and it involves - among other things - a third party's reasonable, but erroneous, assumption that two or more people are carrying on a business for profit. c. Mario and Luigi are both partners of Kingdom Mushrooms, a business registered with the state authorities. Although Mario is an investor and receives a specified share of the profits, he does not have any managerial control of the business. Luigi, unlike Mario, controls the day-to-day operations of the business. Statement: Based on this information, Mario is most likely a limited partner and is not held liable for partnership obligations beyond the amount of his investment, and Luigi is a general partner. d. Steven comes to you, his attorney, for a consultation about choosing an appropriate business entity for his new business with Tom. Though Steven and Tom prefer an informal arrangement, they are willing to balance this objective to limit personal liability. In fact, Steven and Tom say that their sole objective is to limit personal liability. Statement: Steven and Tom can achieve this goal by forming a limited liability company or a corporation, but not by forming a general partnership. e. Adam is a longtime shareholder in McDonalds Corporation, a famous fast food chain. Adam has decided he wants to start a rival fast food chain known as Good Burger. Statement: Adam has the right to check all of McDonalds' books and records, even though he is checking those records with the intent of gaining access to secrets and digging up dirt.

E - False. To inspect the books, a shareholder must meet these 3 requirements: (1) Ownership: 5% (or more) of the stock or at least 1 share for more than six months, • (2) Make a written demand, and • (3) Must have a proper purpose - to check one's investment, see how directors and officers are doing their job. (NOT to demand access to get secrets, or to dig up dirt, or perhaps to press political points). LSB 165. There may be other problems in terms of Adam's being able to inspect McDonald's books and records (has he met the ownership and written demand requirements?); regardless, though, Adam certainly does not seem to have a proper purpose. That alone negates his right to inspect McDonald's books and records.a. Statement: The business judgment rule, if it is to be invoked, is a defense against suits claiming corporate directors and/or officers should be held liable for an allegedly poor decision. • Answer: True. The Business Judgment Rule (BJR) generally means, that as long as directors have made an informed decision and are not interested in the transaction being considered, a court will not question whether the directors' action was wise or whether they made an error of judgment or a business mistake. As a result, directors will be personally liable for loss to the corporation only if the elements of the BJR defense are not satisfied. LSB 173-175; Barron's page 394. • b. Statement: Partnership by estoppel is a form of apparent authority, and it involves - among other things - a third party's reasonable, but erroneous, assumption that two or more people are carrying on a business for profit. • Answer: True; Lectures; LSB 142. Partnership by estoppel is similar to the concept of apparent authority (agency by estoppel) found in agency law. The key question is: Did partners create or permit third parties' reasonable belief and reliance on a partnership [the belief that a person was acting as a partner-agent on behalf of his/her supposed co-owners/partners (the supposed partnership)]? If the answer is "yes," the third party may, in that context, be able to treat the business like a partnership even if it was not an actual partnership (e.g., the so-called partner was actually not a partner and thus not actually able to bind his/her "partners"). • c. Mario and Luigi are both partners of Kingdom Mushrooms, a business registered with the state authorities. Although Mario is an investor and receives a specified share of the profits, he does not have any managerial control of the business. Luigi, unlike Mario, controls the day-to-day operations of the business. • Statement: Based on this information, Mario is most likely a limited partner and is not held liable for partnership obligations beyond the amount of his investment, and Luigi is a general partner. • Answer - True. Limited partners are investors, somewhat similar to corporate shareholders, not managers. "Limited partners contribute capital and receives a special share of the LP's profits." However, limited partners only risk what they put in. In return for risking only their investment, limited partners have no management or control of the business "and are not held personally liable for partnership obligations beyond the amount of his/her capital contribution." LSB • 144. See also Barron's page 328; Lecture. Partnership obligations can best be viewed as meaning the obligations of the partnership itself. (Duties of a particular partner would be called partner obligations or partner duties.) So, when one is a limited partner, one is only liable for debts or liabilities of the partnership (those partnership obligations) up to the amount the limited partner invested in the • limited partnership. That is, one's liability is limited. Without any involvement in the operations of the company past his capital investment, Mario is not considered a general partner, but a limited partner. • d. Steven comes to you, his attorney, for a consultation about choosing an appropriate business entity for his new business with Tom. Though Steven and Tom prefer an informal arrangement, they are willing to balance this objective to limit personal liability. In fact, Steven and Tom say that their sole objective is to limit personal liability. • Statement: Steven and Tom can achieve this goal by forming a limited liability company or a corporation, but not by forming a general partnership. • Answer: True. Although LLCs and corporations provided limited personal liability, general partners are personally liable for debts of the partnership. LSB 138 & 143-144. Partners are also jointly liable for contracts; therefore, any lawsuits based on a contract (including partnership debts) must be brought simultaneously against all partners. • e. Adam is a longtime shareholder in McDonalds Corporation, a famous fast food chain. Adam has decided he wants to start a rival fast food chain known as Good Burger. Statement: Adam has the right to check all of McDonalds' books and records, even though he is checking those records with the intent of gaining access to secrets and digging up dirt. •

. Which of the following is subject to double taxation? a. LLC b. Limited Partnership c. Sole Proprietorship d. S Corporation e. Corporation

E. Barron's pages 325 & 367; LSB 150. In Barron's, Item 19 refers to the "double taxation" of corporations (Barron's page 325). The definition of an S Corporation on Barron's page 367 refers to the "double taxation" of corporate businesses.

. Jeff is a volunteer at a Hospice Thrift Store. Jeff's large nametag clearly states that he is a volunteer. Christian comes into the store and offers Jeff a one-year contract for a food service to be operated out of the store with Hospice getting 20% of the profits and Christian getting 80% of the profits. Jeff is always hungry when he volunteers; so, with a huge laugh and shouting, "Even us volunteers need to eat," Jeff quickly agrees. The manager on duty overhears the exchange, loudly laughs, and does nothing. Hospice refuses to allow Christian to operate a food service in the thrift store. Who is liable for the alleged contract? A. Hospice, for up to 80% of any losses sustained B. Jeff and Hospice, each completely C. Hospice only D. The manager on duty only E. None of the above

E. LSB 107-108. The agent is liable to the third party if the agent lacks authority to make a contract with the third party, but, if it is clear, or should be clear, to the third party that the agent lacks authority, the agent alone is liable. Jeff, as a volunteer, lacks the authority to bind Hospice to a contract. It should be clear to Christian that Jeff lacks the authority because his nametag states he is a volunteer, so only Jeff is liable.

A ___ is a business form in which the owner and the entirety are not legally distinct from each other and taxes are paid at the individual level. A) Partnership B) Limited Liability Company C) Corporation D) Professional Association E) Sole Proprietorship

E. LSB 131-132; Barron's 320-328. The sole proprietorship is the simplest business format in which an individual owns and operates the company, paying taxes at the individual level. The law does not look at the individual as separate from the company. The owner would be personally responsible and potentially have unlimited liability for actions against his company.

. Which of the following scenarios does NOT illustrate one of the advantages of forming a sole proprietorship? A) Meredith is a budding entrepreneur and has a promising business consulting idea. She decides to form a sole proprietorship because of the ease of formation and less stringent requirements. B) Grant was a freelance writer for various newspapers. One day, he decided to start his own online blog with the intention of having exclusive control over the material to be posted on the blog. C) Chloe and Katie used to be in business together and operated a flower shop downtown. Chloe wanted to expand the business to gardening goods and ended up creating her own sole proprietorship to avoid sharing profits. D) Makala is interested in starting her own accounting business for businesses and high net worth individuals. She looked at the requirements for her state, and Makala noticed that she would only need a professional license and did NOT need to file with the state. E) Chris went to school for nutrition management and decided to start his own business as a nutritionist. He decides to form a sole proprietorship, believing that he would benefit from limited liability in this business format.

E. LSB 134; Barron's pages 319-325: A person that forms a sole proprietorship would not be doing so for limited liability purposes as the sole business owner is liable for all actions of the proprietorship. Answer A is incorrect because the ease of formation is one of the advantages of forming as it has few formation requirements. Answer B is incorrect because a perk of forming a sole proprietorship is having exclusive control over the business. Answer C is incorrect because a benefit of forming a sole proprietorship is not having to share in the profits of a business. Answer D is incorrect because many sole proprietorships only require the business owner to have a certain license and is not required to file with the state.

. Fred and Daphne, both of whom despise taxes but do not mind filing papers with government agencies, decide to start a business where they solve various crimes. Fred wants to manage and control all aspects of the business and does not mind being held liable for any given circumstances. However, Daphne does not want to be as involved in the business or as responsible for problems or damages to others; and she only invests a small percentage into the business in exchange for small profits on her investments. Daphne's role can best be described as a(n): a. Limited contractor b. General partner c. Essential stockholder d. Mutual agent e. Limited Partner

E. LSB 144 & Barron's 343. Answer A is incorrect because limited contractors are hired by businesses for a specific task, such as work as a plumber. Answer B is incorrect. General partners play a bigger role in the business than do limited partners, making the general partner more liable for company dealings. LSB 143-144. Answer C is incorrect. Stockholders are characteristic of corporations. Barron's page 753. Answer D is incorrect. Agents make decisions on behalf of their principal, which is not a description of Daphne's role. Barron's page 297.

. Krista currently owns ten shares of stock in Corporation X. This accounts for 10% ownership in Corporation X. Krista has preemptive rights on those shares, which means that if Corporation X decides to sell 100 additional shares, it must do the following: a. Allow Krista to look over its books and records b. Pay dividends to Krista within the business judgment of the board c. Give Krista 10 shares, as a gift d. Allow Krista to sell her shares below market value e. Give Krista the option to purchase 10 shares

E. LSB 165; Barron's page 371; Lectures. Preemptive rights - Allows shareholders to maintain their relative stock ownership strength (their proportion of the total equity) - May be granted to shareholders in the Articles of Incorporation or other corporate documents. So, the corporation cannot sell additional shares outside the corporation without the existing shareholders' permission (or, at least, his right to buy his proportion of the stock).

. Teddy is a shareholder of Chests and More, Inc. He has consistently owned 9% of the company, with these shares valued at $65,000 currently; and Teddy has been a shareholder since the company was founded. Recently, Teddy discovered that Chest and More's officers have been making poor business decisions that put the company further and further into debt. Teddy files a derivative suit. In the complaint, Teddy alleges that a demand for the board of directors to file the case would have been futile because most of its members were officers, too. Is there any issue with this derivative suit? a. Yes. Shareholders cannot bring derivative suits. Only a director can bring a derivative suit. b. Yes.The case would need to be a class action. c. Yes. Teddy owns less than 10% of the shares. d. Yes. Teddy owns less than $100,000 worth of stock. e. No. Teddy followed all the requirements.

E. LSB 166; Barron's 390. As an owner, to sue in a derivative suit, a shareholder must have standing to sue: (1) Own stock throughout the entire litigation, starting when the injury occurred throughout entire time period at issue; (2) Have made a demand to the Board of Directors to bring the suit, unless such a demand would have been futile; and (3) If the shareholder owns less than 5% of the stock or under $50,000 of stock, have posted a bond for legal expenses.

Which of the following acts is most likely to cause a court to pierce the corporate veil? A. Failure to designate a registered agent in the Articles of Incorporation. B. Retention of excess capital. C. Failure to conduct a significant portion of business in the chartering state. D. Failure to hold a monthly shareholder meeting. E. Using corporate assets for the owner's personal purposes.

E. LSB 177. The three main reasons for piercing the corporate veil are misrepresentation, intertwining, and domination. Using corporate assets for personal use would fall under intertwining.

. Axe Capital entered into a contract with Chuck Holdings for the sale of its legal pad manufacturing business. The transaction proved to be ultra vires. Which of the following parties, for the reason stated, may properly assert the ultra vires doctrine? A. Axe Capital to avoid performance. B. Axe Capital to rescind the sale. C. Chuck Holdings to avoid performance. D. Chuck Holdings to rescind the sale. E. A shareholder of Axe Capital to enjoin the sale.

E. LSB 178. Shareholders may assert the ultra vires doctrine, not the corporation itself.

. Yolanda is a director of Smith's Real Estate Sales. The stated purpose of the corporation is the buying and selling of real estate property in Florida. Yolanda enters into a contract for the corporation to buy 50 antique cars. Yolanda thinks the contract will make the corporation a lot of money. A group of shareholders seeks an injunction under the ultra vires doctrine. Is this appropriate? A. No. Only the government can seek an injunction under the ultra vires doctrine. B. No. The group of shareholders did not make a demand on the board first. C. No. Only a disinterested director can seek an injunction under the ultra vires doctrine. D. Yes. Anyone can seek an injunction under the ultra vires doctrine. E. Yes. A group of shareholders can seek an injunction under the ultra vires doctrine.

E. LSB 178. The Latin term ultra vires means "beyond the powers of the corporation," such as by violating the law. Usually, an ultra vires claim centers on the corporate officers' or directors' failure to follow the procedures needed for proper corporate actions. Just two parties can obtain an injunction against ultra vires corporate activity: (1) shareholders - in a suit against the corporation or via a derivative suit; and (2) the government, so A, B, C, and D are incorrect. Answer B might be relevant for a derivative action.

Choose the True statement: a. Three aspects of a corporation are (1) perpetual existence, (2) limited liability for shareholders, and (3) easy transferability of ownership. b. Since Randall holds a significant share in Prestige Worldwide, a large corporation in New York, he has the ability to make business decisions for the corporation, such as ordering materials from its suppliers and requiring dividends to be declared. c. A corporate opportunity may never be taken for personal use by an officer or director of the corporation. d. Both a and b. e. None of the above.

a. Three aspects of a corporation are (1) perpetual existence, (2) limited liability for shareholders, and (3) easy transferability of ownership. Answer: True; LSB 148. b. Since Randall holds a significant share in Prestige Worldwide, a large corporation in New York, he has the ability to make business decisions for the corporation, such as ordering materials from its suppliers and requiring dividends to be declared. Answer: False. LSB 167 ("As a shareholder, a corporate owner cannot carry out managerial functions for the corporation...[a shareholder] cannot purchase stock for corporation, cannot make ordinary business decisions for corporation (hiring and firing of employees, dealing with suppliers, etc.)..."). c. A corporate opportunity may never be taken for personal use by an officer or director of the corporation. Answer: False. There should be full disclosure to, and approval by, disinterested board members. LSB 171-172. d. Both a and b. False. e. None of the above. False.

Choose the true statement: a. If a shareholder loses a derivative action against a corporation, that shareholder must sell his/her shares in the corporation. b. James is the CEO of a successful corporation, "Shoez, Inc." James decides that "Shoez, Inc." will be merged with an acquiring company, "iShoes, Inc." James makes this decision rapidly and without any advice from experts. True or False: It is likely that James will be protected under the business judgment rule. c. Rob wants to start a company so that he can sell photographic equipment. True or False: Rob should choose a sole-proprietorship so he can have exclusive control over any of the investors in his business. d. Shareholders rarely have any significant managerial powers in their roles as shareholders. e. Corporate profits are only taxed if they are distributed to the shareholders.

d. Shareholders rarely have any significant managerial powers in their roles as shareholders. Answer: True. Shareholders have no such duties. LSB 167; Barron's page 384. a. If a shareholder loses a derivative action against a corporation, that shareholder must sell his/her shares in the corporation. Answer: False; Lectures; LSB 166; Barron's page 390. A major risk is that if a judge throws the suit out, the suing shareholder must pay the other side's lawsuit coasts (most significant of all being the attorney fees). b. James is the CEO of a successful corporation, "Shoez, Inc." James decides that "Shoez, Inc." will be merged with an acquiring company, "iShoes, Inc." James makes this decision rapidly and without any advice from experts. True or False: It is likely that James will be protected under the business judgment rule. Answer: False. The Business Judgment Rule only protects directions who act in good faith and with reasonable belief that their conduct legally and legitimately assists in achieving the corporation's purposes. James did not take due care, one of the five elements of the BJR. LSB 174-175; Barron's page 394. Lectures. c. Rob wants to start a company so that he can sell photographic equipment. True or False: Rob should choose a sole-proprietorship so he can have exclusive control over any of the investors in his business. Answer: False. The nature of a sole proprietorship is one investor. LSB 134. e. Corporate profits are only taxed if they are distributed to the shareholders. Answer: False. Corporations are legal "persons" and can be taxed on their income as individuals are. Whether and how much corporations are taxed can be a highly complex matter, and simply to say that - in effect - corporate profits will not be taxed unless distributed to shareholders is erroneous.

. Henry is interested in forming a corporation, T.E.S.T. Co. ("TEST"). He files all the paperwork, submits it, and gets to work. A few months after his corporation has been up and running, one of TEST's distributors does not want to pay. The plaintiff is arguing that he does not have to pay because TEST is not a valid corporation. Henry argues that he has a certificate, so TEST must be valid. Who is correct? a. Henry because TEST is, at the very least, a de facto corporation. b. Henry because TEST constitutes a corporate opportunity. c. Henry because Henry has at least tried to get a corporate certificate, and Henry now must wait. d. Plaintiff because TEST is a de jure corporation. e. Plaintiff because TEST is a benefit corporation.

A. Barron's page 363; LSB 164. The legitimacy of Henry's corporation can only be challenged by the state. Since the plaintiff is only a third party, he must proceed as if TEST is a valid corporate entity. This was also discussed in my review session, too.

. A publicly traded American corporation might choose to be incorporated in Delaware for all the following reasons EXCEPT FOR: A. Delaware places more restrictions on selling or mortgaging assets, and weaker restrictions on merging businesses which makes it easier to fight hostile takeover attempts. B. Delaware provides staggered elections for board members; that way it is more difficult to take over the board in a single election. C. The Courts of Chancery, which is exclusive to Delaware, has special recognition of corporate law needs. D. Delaware provides strong indemnification rights for officers and directors. E. Delaware allows companies to file for bankruptcy in the state of their main office.

A. LSB 153-154. Delaware provides: (1) Few restrictions on selling/mortgaging assets, merging businesses, or fighting takeovers; (2) Strong indemnification rights for officers and directors; (3) Weak rights for dissenting shareholders; (4) Staggered elections for board of directors; (5) Less mandatory disclosure about personnel and purposes; (6) Debtor-friendly courts for corporate bankruptcy cases; and (7) No corporate income tax for corporations formed in the state if they do not transact business in the state. Barron's page 364.

. Mr. Business is a shareholder for a massive company, LMN Enterprises, Inc. ("LMN"), that is incorporated in Delaware and has extensive manufacturing operations, ships goods worldwide, has hundreds of retail stores in North America (mainly the United States), has a large network of "partner" suppliers and licensees, and employs over 40,000 people. Mr. Business notices that the company's profits are declining, and he wants to "shake up" the company. Which of the following will require shareholder approval before LMN acts? a. Decide a new way to operate its stores. b. Acquire stock and therefore combine with a competitor. c. Fire the janitorial staff and hire independent custodial services to do the cleaning at LMN's corporate offices throughout the United States. d. Having a process for filling in for a corporate officer when he/she is unavailable. e. Hiring key employees as managers of LMN's stores.

B. Barron's page 385. In a merger, approval is required by the shareholders of the continuing corporation.

. Barbara, Yijuan, and Omar are starting a candy business. They will specialize in chocolate covered pretzels and strawberries. The business will likely have several locations in both Georgia and Florida. This is the first business for each of these owners/operators, and they would like to limit the paperwork and filing requirements to as little as possible. Since there will be several locations, Barbara, Yijuan, and Omar cannot be everywhere at once; so it is likely that they will hire a manager and staff for each of the locations. They do not like paying taxes and would like to adopt the business format that will result in the best tax situation as possible. What type of business organization would be best? a. sole proprietorship b. general partnership c. limited liability company d. corporation e. limited partnership

B. LSB 138 (The UPA's definition of a general partnership: "An association of two or more persons to carry on as co-owners of a business for profit." Key elements thus are common ownership, sharing of profits/losses, and management rights for all partners); Barron's pages 320-325.

. Which of the following are actions that can be taken by the Board of Directors: I. Full access to the corporate books for a limited amount of time II. Determination of dividend amounts to be given III. The ability to disapprove of a merger or acquisition IV. Altering the corporate bylaws V. Buy, sell, or lease corporate assets a. I, II, IV, V b. II, III, IV, V c. I, III, IV, V d. I, II, III, IV e. All are actions that can be taken by the Board of Director

B. LSB 169. Corporations, subsection Directors, Directors' Powers: BOARDSCAM. While statement I seems correct at first glance, the Board of Directors has full access to the corporate books with "no minimal time or minimal ownership amount".

. In Connecticut, Adam and Peter are fiduciaries for Adam's Bakery and its shareholders. They disclosed a corporate opportunity, without notifying shareholders/the board of directors, to Peter's brother. Since their disclosure was not told ahead of time to disinterested directors, Adam and Peter can be held liable for a breach of , but will not be if they can clearly show that . a. Ultra Vires, the exploited opportunity helped the company b. Corporate Opportunity Doctrine, the exploited opportunity caused no harm to the corporation c. Business Judgment Rule, they will be held liable no matter what d. Ultra Vires, they will be held liable no matter what e. Disinterestedness, the exploited opportunity caused no harm to the corporation

B. LSB 171; Lectures; Barron's page 393. "Directors or officers violate their fiduciary duty if they personally avail themselves of a business opportunity that should have been reserved for the corporation."

. Zach is the general partner of "See Ya Later, Alligator," a limited partnership (LP) that sells Florida Gator apparel. Sam and Jim are limited partners that work for the LP but do not participate in day-to-day management decisions or otherwise run the business. If "See Ya Later, Alligator" goes bankrupt, which partner(s) if any, need to worry that he/they could lose his/their personal belongings to creditors of "See Ya Later, Alligator?" a. Sam and Jim b. Zach c. Zach and Sam, but not Jim because he only works part-time d. All three people (Jim, Sam, and Zach) because they are all partners. e. None of the partners, because people's business and personal belongings are always, as a legal matter, separate.

B. The Tug-of-War Example is given in the Topical Lecture on Partnerships. See LSB 140 & 144. General partners have unlimited legal responsibility, whereas limited partners share the liability for debt. However, limited partners only risk what they put in. In this scenario, Sam and Jim are both limited partners, and Zach is a general partner. Zach would be at risk of losing personal belongings. While answer D does correctly point out that they are all partners, the most important information here is what type of partner they are. Answer E would be correct if the company structure was under a corporation, where the business is a distinct entity separate from the owners.

. Cosmo Kramer recently started his own sole proprietorship business, Cosmo's Convenience Store. Cosmo owns and operates the business and pays taxes at the individual level. Due to mounting debt, he closes the business. Cosmo... a. only has to pay whatever debt he can from the proceeds of liquidating the business. b. only has to pay $10,000 or 50%, whichever is less. c. has to pay all of the debt because he is still personally liable and can even have his personal assets seized to pay the debt. d. is not liable at all because the debt cannot be collected from a business that no longer exists. e. only has to pay debts accrued prior to the final 60 days of operations

C. Barron's pages 320-325. Sole Proprietorships have the disadvantage of unlimited personal liability - personal and business assets are not separate. Personal assets are liable for seizure if the business cannot pay off its debt.

. Which is not a benefit of forming a limited liability company? a. Limited liability for members. b. Avoidance of double taxation. c. Reliance on a well-developed body of case law. d. Ability to have an infinite number of members. e. Informal operation of day-to-day business.

C. LSB 145. ''The law on business management, liability, dissolution, and other matters is still relatively new and unclear, especially compared to more established forms such as corporations and partnerships."

. The Smith-Kind Corporation filed its articles of incorporation with the state but forgot to include one item of information required by statute. Nevertheless, the state overlooked the omission and issued Smith-Kind a corporate charter. Smith-Kind will be operating as a(n): A. Close corporation B. Partnership, joint venture, or franchise (one of these three) C. De facto corporation D. De jure corporation E. Association

C. LSB 164. A de facto corporation exists when there is a glitch in the formation of the corporation. A de facto corporation is treated like a corporation since the incorporators tried in good faith to form the corporation correctly.

. In corporation ABC, there are two boards of directors. It is required that one-half of one of the boards consist of union members. In which country would you most likely find this corporation and what are the names of the two separate boards? a. France; supervisory and lower b. England; supreme and lower c. Germany; supervisory and lower d. France; superior and inferior e. Germany; superior and inferior

C; Lectures; LSB 168. "Under the German system known as co- determination, workers' councils are legally defined bodies that appoint half the members of a company's supervisory board."

. Ken wants to open a business and wants to use the trademarks of a company that is already established. This is an example of: a. General Partnership b. Limited Partnership c. Corporation d. Franchise e. Limited Liability Company

D. Barron's page 348; LSB 134. A franchise is an agreement that allows a person to use the trademark, copyrights, and other property in exchange for a fee and royalties. This arrangement is an essential feature of franchising, while it is not generally relevant to the formation of partnerships, corporations or LLCs, making A B C and E incorrect.

. Brock just got out of Business School. He is trying to form a corporation, but he has no idea where to establish the headquarters of the business. He is flexible and can go anywhere in the United States. He comes to you for advice. Which of the following choices is the best place for Brock form his Corporation? a. Mississippi b. Missouri c. Louisiana d. Delaware e. New York

D. LSB 155-156. Delaware is the number one place to incorporate a corporation. Delaware is (1) pro-management when it comes to Delaware law, and (2) does not have corporate income tax for corporations formed in Delaware as long as they do not transact business in the state. Other states do not compare. Therefore, D is the correct answer.

. Many corporations incorporate in Delaware because: A. Delaware pays promoters a finder's fee B. Delaware does not allow derivative suits C. Delaware corporations need not pay state taxes D. Delaware laws ordinarily do not restrict the powers and actions of management E. Delaware, unlike most other states, even permits the incorporation of businesses that have not yet started to operate

D. LSB 156. Two main reasons for incorporating in Delaware are (1) Freedom for management and (2) Relative certainty in the law

. Suppose that many of the shareholders in Jones Corporation feel like those in control of that Corporation and remedy a wrong suffered by Jones Corporation. Therefore, the shareholders can take action in the form of a... a. Jurisdictional suit b. Class Action suit c. Shareholder suit d. Derivative suit e. Homestead suit

D. LSB 166. A derivative suit is a lawsuit "brought by one or more shareholders on behalf of the corporation and for its benefit. Since these suits are not for the benefit of the suing party, any recovery belongs to the corporation." It arises out of any owner's general power to oversee his/her business. Usually, derivative suits concern alleged poor business judgment (in effect, often acts of negligence) or a breach of corporate opportunity.

. Sylvia is the CFO of Soul Cuts, Inc. In her position as CFO, Sylvia decides to underreport her company's profits in order to save money on the tax bill. By not making an accurate financial report "to the best of his knowledge" She is in violation of which Congressional Act? A. Sherman Act B. Model Business Corporation Act C. Affordable Healthcare Act D. Sarbanes-Oxley Act E. None of the Above

D. LSB 170-171. The Sarbanes-Oxley (SOX) of 2002 provides for disclosures and certifications, auditor independence from firms, stricter controls by government (e.g., the Public Company Accounting Oversight Board, Section 404 ("Internal Control Systems").

. Juana is the chief financial officer of Yanni Inc. Which Congressional act requires officers such as Juana to swear "to the best of her knowledge" that her financial reporting is accurate? a. Model Business Corporation Act b. Private Securities Litigation Reform Act c. Sherman Act d. Sarbanes-Oxley Act e. None of the above

D. LSB 170. The Sarbanes-Oxley Act requires for financial executives to make accurate statements to the best of their knowledge. A) The Model Business Corporation Act attempts to make corporate laws uniform within different states. B) "The Private Securities Litigation Reform Act (PSLRA) is premised on the notion that lawsuits are interfering with efficient capital promotion." LSB 170.

. Jack and Jill are shareholders in "Jeri & Jerry's" corporation. Together Jack and Jill file a shareholder's derivative suit against the directors of "Jeri & Jerry's." Jack and Jill win their lawsuit, and a monetary judgment is issued against the corporate directors. Who receives what the directors have to pay? a. Jack and Jill split the money equally and each receives 50%. b. Jack receives 30% of the money, and Jill receives 70% because Jill paid 70% of the legal fees. c. The money is donated to charity d. The judgment is paid directly to the corporation e. The judgment is paid directly to the corporation, and the corporation must distribute all of the money equally to each shareholder.

D. Lectures; LSB 166. In a derivative suit, any recovery belongs to the corporation.

. Which of the following are required to have an official partnership? I. Written contract II. Implied intent, or another form of intent, to form a partnership III. Express intent to form a partnership IV. Two or more persons carrying on a business for profit a. I & III b. I & II c. III &IV d. II, & IV e. I, II &IV

D. There must be intent, which can be either implied or express. Implied is subsumed within express, but not vice-versa. So implied could be seen as needed, while express need not be. I.e., the general partnership can be non-verbal and certainly need not be in writing. A partnership is an association of two or more persons to carry on as co-owners of a business for profit. They do not need to file anything with the state, and often people have them without even knowing they were established. Thus, there is no need for express intent to form a partnership LSB 138-141. Lectures.

. Which of the following evidently tend to seek the least amount of outside funding and support? a. Corporations b. Silent Partnerships c. Hybrid Business d. Proprietorship e. General Partnership

D; LSB 135. A proprietorship tends to seek the least amount of outside funding because the owner usually is small, and lenders are wary of the owner's unlimited liability. Many partnerships and corporations form for the benefit of having multiple investors.

. Diane, Wilma, and Leigh form a corporation. They are the only three shareholders and directors of the corporation. If Diane dies, the corporation: A. is wound up B. is terminated by law C. is dissolved D. the corporation becomes a limited liability company E. as an entity is generally unaffected

E. Corporations are distinct from the owners, and they can be perpetual. LSB 152-153. B and C would be relevant if the structure of the business was integrated with the owners, such as a sole proprietorship or partnership.

. When two distinct corporate entities enter into a business together to carry out a short-term enterprise, typically for a rather specific (constrained) purpose, the resulting business format is most likely to be a: A. Franchise B. General Partnership C. S Corporation D. Limited Partnership E. Joint Venture

E. LSB 133. Joint ventures are examples of hybrid business categories that entail two parties coming together and collaboratively embarking on a business venture together. The other options are, by definition, incorrect as they entail either one corporation (S Corporation), are not a business format but a relationship (a franchise), or likely multiple individuals (General and Limited Partnerships) that - at the very least - tend to have broader purposes (General and Limited Purposes).

. Hidalgo has an opportunity to join the Board of Directors for a corporation. Hidalgo has been researching the duties of directors. What will be required of Hidalgo should he decide to take the position? a. Decide whether to declare dividends b. Appoint and remove officers c. Create bylaws d. Determine stock prices for newly issued shares e. All of the above are powers of the Board of Directors

E. LSB 169. Directors' powers: BOARDSCAM.

. Which one of the following is not one of the five elements of the Business Judgment Rule? a. No personal Interest (disinterestedness) b. Due Care c. Good Faith d. No Abuse of Discretion e. Unforeseeable Circumstances

E. Lectures; LSB 174-175; Barron's page 376. The five elements of the Business Judgment Rule are 1) business decision making 2) disinterestedness 3) due care 4) Good Faith and 5) no abuse of discretion.

Jane and Elizabeth are on the board of directors of Blue Inc. They were voted in last month during the shareholder meeting. Which of the following is most likely false regarding their directorship? A. They can choose to declare a dividend of $1/share B. They can choose not to declare a dividend C. They have an absolute right to examine the corporate records D. They have the right to approve of a merger with Orange Inc. E. They must be shareholders of Blue Inc.

E. See LSB 168-169. D (Dividends) in BOARDSCAM cover answers A and B. The board can decide whether to declare or not. For Answer C, see R-Records in BOARDSCAM. The Board has an absolute right to inspect corporate records. For Answer D, there is A- Acquisitions (and mergers) in BOARDSCAM. For Answer E, see LSB 168 -in the section under "directors," the LSB states that directors often are shareholders but are not required to be.

. Choose the True Statement: • a. For every corporation, you must have a state charter, which is granted after you file a constitution with the state. This charter is considered the corporation's birth certificate and is usually constructed narrowly to fit the corporation's needs. • b. China's agency and partnership law is more similar to that of the United States than is the agency and partnership law of Mexico. • c. Lisa and Alyson are among some of the promoters for a corporation, Widgets R US, which has not yet come into existence. True or False: Because individuals are considered a separate entity from the corporation, Lisa and Alyson are always protected from actions they take in regards to Widgets R US, both before and after the corporation is formed. • d. Kane and Abel wanted to start a car repair business. Kane is the "go to" guy in car repairs and management. Abel knows nothing about car repairs but has the finance to support the business. To limit Abel's liability, Kane and Abel enter into a limited partnership. True or False: As a general partner, Abel will only be liable for the amount he invests in the company. • e. Jimmy and Chad decided to start an ice skate rental company. Jimmy is the brains behind the operation and knows a great deal about ice skating. Chad does not know much about the ice skate rental industry but does have the money to fund the business. To limit Chad's liability, Jimmy and Chad enter into a limited partnership. True or False: As a general partner, Chad will only be liable for the amount he invests in the business. •

b. China's agency and partnership law is more similar to that of the United States than is the agency and partnership law of Mexico. Answer: True. LSB 136-137; Lecture example discussing countries such as China, Mexico, and Morocco). a. For every corporation, you must have a state charter, which is granted after you file a constitution with the state. This charter is considered the corporation's birth certificate and is usually constructed narrowly to fit the corporation's needs. Answer: False. The charter (sometimes called the articles of incorporation, or articles) is the grant of corporate existence, the birth certificate of the corporation. However, so that the corporation has flexibility and does not have to amend and re-file its Articles of Incorporation; keep its articles of incorporation very large and flexible. LSB 162-163; Lectures. c. Lisa and Alyson are among some of the promoters for a corporation, Widgets R US, which has not yet come into existence. True or False: Because individuals are considered a separate entity from the corporation, Lisa and Alyson are always protected from actions they take in regards to Widgets R US, both before and after the corporation is formed. Answer- False; LSB 162; Lectures. In the course of forming the corporation, the promoter may incur costs, make contracts, and do other acts [e.g., lines up employees and suppliers] in furtherance of the corporation." Because the corporate entity is not yet created, the promoter is potentially liable for his actions - not protected by a corporate shield. d. Kane and Abel wanted to start a car repair business. Kane is the "go to" guy in car repairs and management. Abel knows nothing about car repairs but has the finance to support the business. To limit Abel's liability, Kane and Abel enter into a limited partnership. True or False: As a general partner, Abel will only be liable for the amount he invests in the company. Answer: False; LSB 144. Only limited partners in a limited partnership are not held personally liable for partnership obligation beyond the amount of his/her capital contribution. e. Jimmy and Chad decided to start an ice skate rental company. Jimmy is the brains behind the operation and knows a great deal about ice skating. Chad does not know much about the ice skate rental industry but does have the money to fund the business. To limit Chad's liability, Jimmy and Chad enter into a limited partnership. True or False: As a general partner, Chad will only be liable for the amount he invests in the business. Answer: False. LSB 144. Limited partners are not held liable for partnership obligations beyond the amount he/she invests. General partners do not benefit from limited liability and may be held accountable for their partner's actions.


Ensembles d'études connexes

Systems of Equations Graphing method

View Set

C727 - Cybersecurity Management I - Strategic

View Set

Good Clinical Practice Course (Medical Clinical Research)

View Set

[Lección 2] Estructura 4.3 - Mi universidad

View Set